You are on page 1of 10

Universidad Nacional

Federico Villarreal
Profesionales formando profesionales
Control de la Calidad
2014
UNIVERSIDAD NACIONAL FEDERICO VILARREAL

1

1.- para n = 200, c = 4, AQL = 0.5% y LTPD = 4%, encuentre los valores de .
Solucin:
AQL = 0.5% = 0.005
LTPD = 4% = 0.04
?
= 0.004 = 0.4%
= 0.100 = 10%

2.- usted es responsable de las compras de cojinetes para el departamento de mantenimiento de
una importante aerolnea. Los cojinetes son fabricados bajo contrato de un proveedor local y
usted deber idear un plan de muestreo de aceptacin adecuado para el caso. La gerencia ha
dispuesto en el contrato correspondiente que el nivel de calidad aceptable es de un 1% de
defectos. Adems, lo proporcin defectuosa tolerable en el lote es el 4%, el riesgo del producto es
de un 5% y del riesgo del consumidor es de 10%.
a) Especifique un plan de muestreo de aceptacin apropiado que satisfaga todos estos
criterios.
b) Dibuje la curva OC correspondiente a su plan. Cul es el riesgo del producto resultante?
c) Determine el AOQL para su plan. Supongamos que el tamao del lote es de 3000.

Solucin:
AQL = 0.01
LTPD = 0.04
= 0.05 Pa = 0.95
= 0.10 Pa = 0.1



P np Pa
AQL 0.005 1 0.996
LTPD 0.04 8 0.100
UNIVERSIDAD NACIONAL FEDERICO VILARREAL

2


a)





b) n =265 ; c =6

Grafica CURVA OC:


Entonces:
= 0.097
= 0.02


c) AOQL=

; para N = 3000.

AQL LTPD
C np n np n
0 0.05 5 2.4 60
1 0.35 35 4 100
2 0.80 80 5.4 135
3 1.3 130 6.8 170
4 1.9 190 8 200
5 2.6 260 9.4 135
6 3.2 320 10.6 165
7 3.8 380 11.8 295
8 4.6 460 13 325
9 5.4 540 14.2 355
10 6 600 15.4 385
p np


0.01 2.65 0.980
0.02 5.3 0.71
0.03 7.95 0.313
0.04 10.6 0.097
0.05 13.25 0.082
0.06 15.9 0.005
0.07 18.55 0.001
-0.2
0
0.2
0.4
0.6
0.8
1
1.2
0 2 4 6 8
Series1
UNIVERSIDAD NACIONAL FEDERICO VILARREAL

3


AOQL = 0.0131=1.31%






3.- la Sun Silk Shampoo Company compra las etiquetas que pega en cada una de las botellas de
champ que vende. Esas etiquetas ostentan el logotipo de la compaa, el nombre del producto y
las instrucciones para el uso del mismo. A veces, la impresin que aparece sobre la etiqueta esta
borrosa o los colores no son los correctos. La compaa desea disear un plan de muestreo de
aceptacin para los elementos que compra. El nivel de calidad aceptable es de 5 etiquetas
defectuosas por cada 500, y la proporcin defectuosa tolerable es el lote es de 5%. La gerencia
desea limitar el riesgo del productor a 5% o menos, y el riesgo del consumidor a 10% o menos.
a. Especifique un plan que satisfaga esos deseos.
b. Cul es la probabilidad de que un embarque que contenga un 3% de defectos, sea rechazado
por el plan?
c. Determine el AOQL correspondiente a su plan. Supongamos que el tamao del lote es de 2000
etiquetas.
Solucin:
a. Para AQL = 1%, LTPD = 5%, = 5% y = 10%, por el apndice Probabilidades acumulativas
de Poisson se procede por tanteo, para encontrar un plan de muestreo.
Si n = 135 y c = 3
np = 135(0.01) = 1.35 ------> = 0.049
np = 135(0.05) = 6.75 ------> = 0.096




P


0.01 0.0089
0.02 0.0131
0.03 0.0086
0.04 0.0035
0.05 0.001
0.06 0.000
0.07 0.0006
UNIVERSIDAD NACIONAL FEDERICO VILARREAL

4
b. Tabla
proporcin
defectuosa (p)
np
probabilidad
de c defectos
(Pa)
0.01 (AQL) 1.35 0.951
0.02 2.70 0.714
0.03 4.05 0.431
0.04 5.40 0.213
0.05 (LTPD) 6.75 0.096
0.06 8.10 0.040
0.07 9.45 0.016
0.08 10.80 0.006
0.09 12.15 0.002
0.10 13.50 0.001

De acuerdo con la tabla, la probabilidad de aceptar un lote con un 3% de defectuosos es
de 0.431. Por lo tanto, la probabilidad de que un lote con 3% de defectos sea rechazado es
de 0.569.

c. AOQL, N = 2000 etiquetas


proporcin
defectuosa (p)
np
probabilidad
de c defectos
(Pa)
AOQ
0.01 (AQL) 1.35 0.951 0.009
0.02 2.70 0.714 0.013 (AOQL)
0.03 4.05 0.431 0.012
0.04 5.40 0.213 0.008
0.05 (LTPD) 6.75 0.096 0.004
0.06 8.10 0.04 0.002
0.07 9.45 0.016 0.001
0.08 10.80 0.006 0.000
0.09 12.15 0.002 0.000
0.10 13.50 0.001
0.000
UNIVERSIDAD NACIONAL FEDERICO VILARREAL

5

4. su compaa suministra jeringas estriles a un distribuidor de artculos para hospitales. En el
contrato correspondiente est estipulado que la calidad no deber ser inferior al 0.1% de defectos,
es decir, 10 partes en 10,000. Durante las negociaciones, usted se entr de que el distribuidor
aplicar un plan de muestreo de aceptacin con n= 350 para determinar la calidad del lote.
a. si el riesgo del productor no debe ser mayor que el 5%, Cul es el nmero de aceptacin ms
bajo, c, que se deber usar?
b. en el proceso de produccin de jeringas se producen en promedio, 17 partes defectuosas por
cada 10,000. Con n=350 y el nivel de aceptacin sugerido en la parte (a), cul es la probabilidad
de que un embarque le vaya a ser devuelto?
c. supongamos que usted desea tener una posibilidad menor del 5% de que su embarque le sea
devuelto. A partir de los datos de la parte (b), qu nmero de aceptacin, c, habra sugerido
usted en la parte (a)? Cul es el riesgo del productor correspondiente a ese plan?
Solucin:
proporcin
np
1 2 3 4 8 6 5 7
probabilidades de c defectos o menos
defectuosa
0 0 1 1 1 1 1 1 1 1
0.01 3.50 0.1359 0.3208 0.5366 0.7254 0.9901 0.9347 0.8576 0.9733
0.02 7.00 0.0073 0.0296 0.0818 0.173 0.7291 0.4497 0.3007 0.5987
0.03 10.50 0.0003 0.0018 0.0071 0.0211 0.2794 0.1016 0.0504 0.1785
0.04 14.00 0.0000 0.0001 0.0005 0.0018 0.0621 0.0142 0.0055 0.0316
0.05 17.50 0.0000 0.0000 0.0000 0.0001 0.0095 0.0015 0.0005 0.004
0.06 21.00 0.0000 0.0000 0.0000 0.0000 0.0011 0.0001 0.0000 0.0004
0.07 24.50 0.0000 0.0000 0.0000 0.0000 0.0001 0.0000 0.0000 0.0000
0.08 28.00 0.0000 0.0000 0.0000 0.0000 0.0000 0.0000 0.0000 0.0000
0.09 31.50 0.0000 0.0000 0.0000 0.0000 0.0000 0.0000 0.0000 0.0000
0.1 35.00 0.0000 0.0000 0.0000 0.0000 0.0000 0.0000 0.0000 0.0000



UNIVERSIDAD NACIONAL FEDERICO VILARREAL

6


c

1 2 3 4 8 5 6 7
a 86% 68% 46% 27% 21% 14% 7% 3%
b

a.- Vemos que el nmero de aceptacin ms bajo es el C=7 debido a que a partir de ah el nmero
es menor que 5.
5. un comprador de componentes electrnicos ha exigido que la proporcin defectuosa tolerable
en el lote sea de 20 partes en 5000, con un riesgo de consumidor de 15%, si el comprador ha
decidido muestrear 1500 de los componentes recibidos en cada embarque, qu nmero de
aceptacin, deseara dicho comprador? Cul es el riesgo del productor si el AQL es de 10 partes
por 5000?
Solucin:
LTPD =0.4%; n=1500; = 15%
a) Np= 1500 x 0.004 = 6 c=2 (con 0.062)

b) AQL=0.02% c=3 np= 1500x0.002=3 = 1 0.0353= 0.577 = 58%


6. Considere el caso de cierta prima para la cual se requiere un plan de un solo muestreo enfocado
en atributos. El AQL es de 1% y el LTD es de 4%. Se han propuesto dos planes. Bajo el plan 1, n=150
y c=4; bajo el plan 2, n=300 y c=8. Son equivalentes estos dos planes? Fundamente su respuesta
determinando el riesgo del productor y el riesgo del consumidor para cada plan.






AQL= 1% ; LTPD= 4%
PLAN 1 n= 150 ; c=4 PLAN 2 n= 300 ; c=8
Np= 150 x 0.01 =1.5 ; = 1.9%
Np= 150 x 0.04 =6 ; = 28.5%

Np= 300 x 0.01 =3 ; = 0.4%
Np= 300 x 0.04 =12 ; = 15.5%

UNIVERSIDAD NACIONAL FEDERICO VILARREAL

7

Como podemos apreciar los planes no son equivalentes. Al aumentar la muestra aumenta el riesgo
del consumidor como del productor.

7. Actualmente usted tiene un plan de muestreo de aceptacin en el cual n=40 y c=1, pero el
rendimiento no le parece satisfactorio. El AQL es de 1% y la LTPD es de 5%-
a) Cules son los riesgos del productor y del consumidor cuando se aplica este plan?
AQL= 1% ; LTPD= 5% ; n=40 ; c=1
Np= 0.4 = 6.2%
Np= 2 = 40.6%

c) Manteniendo la misma razn 1:40 para c:n, incremente c y n para encontrar un plan de
muestreo que reduzca el riesgo del productor a 5% o menos, y el riesgo del consumidor a
10% o menos. Qu riesgos del productor y del consumidor estn asociados a este nuevo
plan?


c) Compare el AQL correspondiente a su plan con el viejo plan. Suponga un tamao de lote de
1000 unidades.
n c p np Pa AOQL

ANTIGUA


40

1
0.01 0.4 0.938 0.01563
0.05 2 0.406 0.03383
c n AQL (0.01)
%
LTPD (0.05)
%
2 80 4.7 9.2
UNIVERSIDAD NACIONAL FEDERICO VILARREAL

8

NUEVA


80

2
0.01 0.8 0.953 0.01588
0.05 4 0.092 0.00767


8-.Para AQL = 1% LTPD= =4% y n =40 Qu valor(es) del nmero de aceptacin c, produciran el
resultado de que tanto el riesgo del productor como el riesgo del consumidor fueron menores de
5%?

AQL = 1% ; LTPD=4% ;n = 400





9-. Para un AQL = 1% ; c= 2 Cul es el mayor valor de n que dar como resultado un riesgo del
productor de 5%
Usando ese tamao de muestra, determine el riesgo del consumidor cuando el LTPD =2%
np
1.6 0.9474
0.8 0.953
0.85 0.9845

0.01n = 1.6
N = 160
0.01n = 0.8
N = 80
c % %
8 2.1 2.2
9 0.8 4.3
UNIVERSIDAD NACIONAL FEDERICO VILARREAL

9
0.01n = 0.85
N = 85

N= 160; 0.02 * 160 = 3.2 = 3
proporcin defectuosos np Pa
3 4.8 0.143

Por lo tanto = 14.3%


10.- Para c = 10 y LTPD = 5%, qu valor de n resulta en un riesgo del consumidor de 5%?
Solucin:
LTPD = 5% y = 5%
np = n * (0.05)
Procedemos a mirar la tabla acumulativa de Poisson para un valor c = 10, que resulte un valor
de 0.05.
Nos da como resultado un valor np entre 17 y 16.8. Por lo tanto, procedemos a tabular:
(


)
np = 16.96
n = 16.96/0.05
Rpta: n = 339

You might also like